IMO 2014/1

Let \( a_0 < a_1 < a_2 \ldots \) be an infinite sequence of positive integers. Prove that there exists a unique integer \( n \geq 1 \) such that

an<a0+a1++annan+1. a_n < \frac{a_0 + a_1 + \ldots + a_n } { n} \leq a_{n+1} .

#Algebra

Note by Calvin Lin
6 years, 11 months ago

No vote yet
1 vote

  Easy Math Editor

This discussion board is a place to discuss our Daily Challenges and the math and science related to those challenges. Explanations are more than just a solution — they should explain the steps and thinking strategies that you used to obtain the solution. Comments should further the discussion of math and science.

When posting on Brilliant:

  • Use the emojis to react to an explanation, whether you're congratulating a job well done , or just really confused .
  • Ask specific questions about the challenge or the steps in somebody's explanation. Well-posed questions can add a lot to the discussion, but posting "I don't understand!" doesn't help anyone.
  • Try to contribute something new to the discussion, whether it is an extension, generalization or other idea related to the challenge.
  • Stay on topic — we're all here to learn more about math and science, not to hear about your favorite get-rich-quick scheme or current world events.

MarkdownAppears as
*italics* or _italics_ italics
**bold** or __bold__ bold

- bulleted
- list

  • bulleted
  • list

1. numbered
2. list

  1. numbered
  2. list
Note: you must add a full line of space before and after lists for them to show up correctly
paragraph 1

paragraph 2

paragraph 1

paragraph 2

[example link](https://brilliant.org)example link
> This is a quote
This is a quote
    # I indented these lines
    # 4 spaces, and now they show
    # up as a code block.

    print "hello world"
# I indented these lines
# 4 spaces, and now they show
# up as a code block.

print "hello world"
MathAppears as
Remember to wrap math in \( ... \) or \[ ... \] to ensure proper formatting.
2 \times 3 2×3 2 \times 3
2^{34} 234 2^{34}
a_{i-1} ai1 a_{i-1}
\frac{2}{3} 23 \frac{2}{3}
\sqrt{2} 2 \sqrt{2}
\sum_{i=1}^3 i=13 \sum_{i=1}^3
\sin \theta sinθ \sin \theta
\boxed{123} 123 \boxed{123}

Comments

Let's call Sk=a0+a1+...+ak S_k = a_0 + a_1 + ... + a_k and suppose Skk>ak+1  mk \frac { { S }_{ k } }{ k } >{ a }_{ k+1 } \; \forall m \leq k Then we have kSk+Sk>kak+1+kSkSkk>Sk+1k+1Sk+1k+1<Skk<Sk1k1<...<S1=a0+a1 k\cdot { S }_{ k }+{ S }_{ k }>k\cdot { a }_{ k+1 }+k\cdot{ S }_{ k }\quad \Leftrightarrow \quad \frac { { S }_{ k } }{ k } >\frac { { S }_{ k+1 } }{ k+1 }\\ \quad \Leftrightarrow \quad \frac { { S }_{ k+1 } }{ k+1 } <\frac { { S }_{ k } }{ k } <\frac { { S }_{ k-1 } }{ k-1 } <...<{ S }_{ 1 }={ a }_{ 0 }+{ a }_{ 1 } Hence, we prove by induction 2(a0+a1)>a0+a1+a2a0+a1>a2kak+1<Sk<k(a0+a1)ak+1<(a0+a1) 2(a_{ 0 }+{ a }_{ 1 })>{ a }_{ 0 }+{ a }_{ 1 }+{ a }_{ 2 }\quad \Leftrightarrow \quad { a }_{ 0 }+{ a }_{ 1 }>{ a }_{ 2 }\\ k{ a }_{ k+1 }<{ S }_{ k }<k(a_{ 0 }+{ a }_{ 1 })\quad \Leftrightarrow \quad { a }_{ k+1 }<(a_{ 0 }+{ a }_{ 1 }) Because a0+a1 a_{ 0 }+{ a }_{ 1 } is a sum of integer number, there must be a unique number in the sequence for which aq+1a0+a1 a_{q+1} \geq a_{ 0 }+{ a }_{ 1 } and aq<a0+a1 a_{q} < a_{ 0 }+{ a }_{ 1 } . We now show that q q is the required number. We have alredy proved that Sqqaq+1 \frac { { S }_{ q } }{q } \leq { a }_{ q+1 } by contraddiction! Thus we have only to show that aq<Sqqqaqaq<Sqaqaq<Sq1q1{ a }_{ q }<\frac { S_{ q } }{ q } \quad \Leftrightarrow \quad q{ \cdot a }_{ q }-a_{ q }<S_{ q }-a_{ q }\quad \Leftrightarrow \quad { a }_{ q }<\frac { S_{ q-1 } }{ q-1 } And because Sq1(q1)+aq(q1)<Sq1+Sq1(q1)Sqq<Sq1q1Sqq<Sq1q1<Sq2q2<...<S1=a0+a1 S_{ q-1 }(q-1)+{ a }_{ q }(q-1)<S_{ q-1 }+S_{ q-1 }(q-1)\quad \Leftrightarrow \quad \frac { S_{ q } }{ q } <\frac { S_{ q-1 } }{ q-1 }\\ \quad \Leftrightarrow \quad \frac { S_{ q } }{ q } <\frac { S_{ q-1 } }{ q-1 } <\frac { S_{ q-2 } }{ q-2 } <...<S_{ 1 }={ a }_{ 0 }+{ a }_{ 1 } Excactly as done beofre we must have aq<a0+a1 a_q < a_0 + a_1 . That is true. And we are done.

Andrea Gallese - 6 years, 11 months ago

Log in to reply

Beautiful solution

Scrub Lord - 4 years ago

Question 1 was way easier than I expected it to be. Bummed that there were no proper number theory problems.

Mursalin Habib - 6 years, 11 months ago

Log in to reply

In recent years, they have made question 1 (and sometimes 4) pretty easy and direct to approach, so that all of the participants (who come from a wide range of ability level) have something to do, and have a chance of geting a HM.

Calvin Lin Staff - 6 years, 11 months ago

This was indeed very nice, but also pretty easy, even for an IMO P1. I have posted my solution in AoPS.

http://www.artofproblemsolving.com/Forum/viewtopic.php?p=3542324&#p3542324

Sreejato Bhattacharya - 6 years, 11 months ago

Here's a conjecture:

The last possible nn that satisfies an<a0+a1++anna_n < \dfrac{a_0+a_1+\cdots +a_n}{n} also satisfies a0+a1++annan+1\dfrac{a_0+a_1+\cdots +a_n}{n}\le a_{n+1}. This is just a guess though.

These later posts will be what I have came up with.

Daniel Liu - 6 years, 11 months ago

Log in to reply

Update: letting a0=xa_0=x and a1=ya_1=y, we find that in order for ak<a0+a1++akka_k < \dfrac{a_0+a_1+\cdots +a_k}{k}, we must have ak<x+ya_k < x+y. However, since x+yx+y is finite, and the sequence is strictly increasing, we must have that there is a maximum kk that satisfies this condition.

So let kk be the maximum such that it still satisfies the condition. Thus, ak+1x+ya_{k+1} \ge x+y. However, a0+a1++akk<x+y+(x+y)++(x+y)k=x+y\dfrac{a_0+a_1+\cdots +a_k}{k} < \dfrac{x+y+(x+y)+\cdots +(x+y)}{k}=x+y. Thus, a0+a1++akk<ak+1\dfrac{a_0+a_1+\cdots +a_k}{k} < a_{k+1}.

This seems really sketchy though. Something seems wrong... one of my inequalities was strict when forming the right inequality, but the inequality in the problem is not strict...

EDIT: found out why mine was strict and the problem's wasn't. As long as the maximum kk satisfies k2k\ge 2, then the upper bound inequality is strict, just like my result. However, the special case of the maximum kk is k=1k=1 yields a non-strict inequality.

TO DO: prove uniqueness of satisfying both inequalities.

Daniel Liu - 6 years, 11 months ago

Log in to reply

I realize that I have to tighten the bound for each ak<x+ya_k < x+y to something more strict, or something like that in order to prove the uniqueness of that one kk that satisfies both inequalities. I feel like I'm almost there, but just can't get over the last little bump. I need to somehow use the ak<ak+1a_k < a_{k+1} to my advantage...

Daniel Liu - 6 years, 11 months ago
×

Problem Loading...

Note Loading...

Set Loading...